Difference between revisions of "2000 AMC 12 Problems/Problem 6"

(Solution)
m (Solution 2)
Line 41: Line 41:
 
First, since all primes in between 4 and 18 are odd, the product is odd. This eliminates options B and D. So <math>21=3*7</math>, but that's not possible. <math>119 = 7 * 17</math>, which works perfectly so the answer is just <math>\boxed{C}</math>.  
 
First, since all primes in between 4 and 18 are odd, the product is odd. This eliminates options B and D. So <math>21=3*7</math>, but that's not possible. <math>119 = 7 * 17</math>, which works perfectly so the answer is just <math>\boxed{C}</math>.  
  
*Note: this problem did not require the strategy used in Solution 1. It takes up to much of your needed time during the test. In another case, it will probably be more usefull.
+
*Note: this problem did not require the strategy used in Solution 1. It takes up too much of your needed time during the test. In another case, it will probably be more useful.
  
 
== See also ==
 
== See also ==

Revision as of 09:59, 12 December 2009

Problem

Two different prime numbers between $4$ and $18$ are chosen. When their sum is subtracted from their product, which of the following numbers could be obtained?

$\mathrm{(A) \ 21 } \qquad \mathrm{(B) \ 60 } \qquad \mathrm{(C) \ 119 } \qquad \mathrm{(D) \ 180 } \qquad \mathrm{(E) \ 231 }$

Solution 1

Let the primes be $p$ and $q$.

The problem asks us for possible values of $K$ where $K=pq-p-q$

Using Simon's Favorite Factoring Trick:

$K+1=pq-p-q+1$

$K+1=(p-1)(q-1)$

Possible values of $(p-1)$ and $(q-1)$ are:

$4,6,10,12,16$

The possible values for $K+1$ (formed by multipling two distinct values for $(p-1)$ and $(q-1)$) are:

$24,40,48,60,64,72,96,120,160,192$

So the possible values of $K$ are:

$23,39,47,59,63,71,95,119,159,191$

The only answer choice on this list is $119 \Rightarrow C$

Note: once we apply the factoring trick we see that, since $p-1$ and $q-1$ are even, $K+1$ should be a multiple of $4$.

These means that only $119 \Rightarrow C$ and $231 \Rightarrow E$ are possible.

We can't have $(p-1) \cdot (q-1)=232=2^3\cdot 29$ with $p$ and $q$ below $18$. Indeed, $(p-1) \cdot (q-1)$ would have to be $2 \cdot 116$ or $4 \cdot 58$.

But $(p-1) \cdot (q-1)=120=2^3\cdot 3 \cdot 5$ could be $2 \cdot 60,4 \cdot 30,6 \cdot 20$ or $10 \cdot 12.$ Of these, three have $p$ and $q$ prime, but only the last has them both small enough. Therefore the answer is $C$.

Solution 2

First, since all primes in between 4 and 18 are odd, the product is odd. This eliminates options B and D. So $21=3*7$, but that's not possible. $119 = 7 * 17$, which works perfectly so the answer is just $\boxed{C}$.

  • Note: this problem did not require the strategy used in Solution 1. It takes up too much of your needed time during the test. In another case, it will probably be more useful.

See also

2000 AMC 12 (ProblemsAnswer KeyResources)
Preceded by
Problem 5
Followed by
Problem 7
1 2 3 4 5 6 7 8 9 10 11 12 13 14 15 16 17 18 19 20 21 22 23 24 25
All AMC 12 Problems and Solutions